Difference between revisions of "1950 AHSME Problems/Problem 22"

Line 5: Line 5:
 
<math> \textbf{(A)}\ 30\%\qquad\textbf{(B)}\ 15\%\qquad\textbf{(C)}\ 72\%\qquad\textbf{(D)}\ 28\%\qquad\textbf{(E)}\ \text{None of these} </math>
 
<math> \textbf{(A)}\ 30\%\qquad\textbf{(B)}\ 15\%\qquad\textbf{(C)}\ 72\%\qquad\textbf{(D)}\ 28\%\qquad\textbf{(E)}\ \text{None of these} </math>
  
==Solution==
+
==Solution 1 (Kind of Lame)==
 
Without loss of generality, assume something costs <math>100</math> dollars. Then with each successive discount, it would cost <math>90</math> dollars, then <math>72</math> dollars. This amounts to a total of <math>28</math> dollars off, so the single discount would be <math>\boxed{\mathrm{(D)}\ 28\%.}</math>
 
Without loss of generality, assume something costs <math>100</math> dollars. Then with each successive discount, it would cost <math>90</math> dollars, then <math>72</math> dollars. This amounts to a total of <math>28</math> dollars off, so the single discount would be <math>\boxed{\mathrm{(D)}\ 28\%.}</math>
  
 +
==Solution 2 (Technical)==
 +
 +
Let the object cost <math>x</math> dollars. After the <math>10%</math> discount, it's worth <math>(1-10%)x=0.9x</math> dollars. After a <math>20%</math> discount on that, it's worth <math>(1-20%)(0.9x)=0.72x</math>. Say the single discount is of <math>k</math>. Then <math>(1-k)x=0.72x</math>. So <math>k=0.28</math>, or <math>k=28%</math>. So select <math>\boxed{D}</math>.
 +
 +
~hastapasta
 
==See Also==
 
==See Also==
  

Revision as of 12:11, 31 March 2022

Problem

Successive discounts of $10\%$ and $20\%$ are equivalent to a single discount of:

$\textbf{(A)}\ 30\%\qquad\textbf{(B)}\ 15\%\qquad\textbf{(C)}\ 72\%\qquad\textbf{(D)}\ 28\%\qquad\textbf{(E)}\ \text{None of these}$

Solution 1 (Kind of Lame)

Without loss of generality, assume something costs $100$ dollars. Then with each successive discount, it would cost $90$ dollars, then $72$ dollars. This amounts to a total of $28$ dollars off, so the single discount would be $\boxed{\mathrm{(D)}\ 28\%.}$

Solution 2 (Technical)

Let the object cost $x$ dollars. After the $10%$ (Error compiling LaTeX. Unknown error_msg) discount, it's worth $(1-10%)x=0.9x$ (Error compiling LaTeX. Unknown error_msg) dollars. After a $20%$ (Error compiling LaTeX. Unknown error_msg) discount on that, it's worth $(1-20%)(0.9x)=0.72x$ (Error compiling LaTeX. Unknown error_msg). Say the single discount is of $k$. Then $(1-k)x=0.72x$. So $k=0.28$, or $k=28%$ (Error compiling LaTeX. Unknown error_msg). So select $\boxed{D}$.

~hastapasta

See Also

1950 AHSC (ProblemsAnswer KeyResources)
Preceded by
Problem 21
Followed by
Problem 23
1 2 3 4 5 6 7 8 9 10 11 12 13 14 15 16 17 18 19 20 21 22 23 24 25 26 27 28 29 30 31 32 33 34 35 36 37 38 39 40 41 42 43 44 45 46 47 48 49 50
All AHSME Problems and Solutions

The problems on this page are copyrighted by the Mathematical Association of America's American Mathematics Competitions. AMC logo.png